14
$\begingroup$

I am not exactly a group theorist, so this may be well-known.

Let $G$ be a finitely generated group such that the cardinality of minimal generating sets of $G$ is bounded above. Does it follow that $G$ is finite?

This is true if $G$ is abelian, but I have no idea about the general case.

$\endgroup$
3
  • $\begingroup$ For groups, I don't know. For general algebras, here is an infinite example. Take an algebra with an injective and nonsurjective map u(x), consider a subalgebra S including and generated by an element b, and add to the unary operation u the unary operation c which satisfies c(x)=b. Then S with the operations u and c has minimal generating sets of size 1. (So does S with u and without c, but that is a little harder to see.) Even though S is not a group, it inspires me to hope there are infinite groups with such bounds. Gerhard "General Algebraic Desires Spring Eternal" Paseman, 2014.01.13 $\endgroup$ Jan 13, 2014 at 21:00
  • 2
    $\begingroup$ Wouldn't an infinite group whose Frattini subgroup of finite index be an example? $\endgroup$ Jan 13, 2014 at 21:26
  • 5
    $\begingroup$ A Tarski Monster is certainly a counterexample. $\endgroup$
    – Derek Holt
    Jan 13, 2014 at 21:35

3 Answers 3

20
$\begingroup$

Just to expand my comment, a Tarski Monster is an infinite group in which, for some fixed prime $p$, all proper nontrivial subgroups have order $p$. It was proved by Olshanskii in 1979 that they exist for all primes $p>10^{75}$.

A set of $p+1$ distinct elements of such a group cannot all lie in the same subgroup of order $p$, so they must generate the whole group, and so we have an upper bound of $p+1$ on the size of minimal generating sets.

It would interesting to know whether there are any less exotic examples.

$\endgroup$
2
  • 6
    $\begingroup$ Actually, any (inclusion-)minimal generating set of a Tarski monster consists of two elements (if $p$ is prime). $\endgroup$ Jan 14, 2014 at 1:38
  • $\begingroup$ Yes of course!! $\endgroup$
    – Derek Holt
    Jan 14, 2014 at 8:54
14
$\begingroup$

There is a simpler construction indeed, namely the Grigorchuk group $\Bbb G$. Though still not very easy, it's definitely "less exotic". For a readable introduction, see our mini-survey, or even better Ch. VIII in de la Harpe's monograph.

Specifically, on your question, it is known that every generating set with 4 elements is redundant. This follows immediately from Pervova's computation of the Frattini subgroup $\Phi(\Bbb G)$ given in this paper. She proved that all maximal subgroups of $\Bbb G$ have index 2 in $\Bbb G$. This easily implies $$\Bbb G/\Phi(\Bbb G) = \Bbb G/[\Bbb G,\Bbb G]= \Bbb Z_2^3.$$
For more on the context of Pervova's work and other results on the Grigorchuk group, see this survey by Grigorchuk (esp. $\S 6$).

It is also worth noting that you shouldn't expect much easier examples. If even one minimal generating set has an element $g$ of infinite order, one can use powers of $g$ to make large minimal generating sets. Unsurprisingly, all elements in $\Bbb G$ have finite orders, which are in fact powers of 2.

$\endgroup$
1
  • 1
    $\begingroup$ The Grigorchuk group is certainly a lot easier to construct and understand than Tarski Monsters. You can cover the former in an advanced undergraduate lecture, but I don't know of many people who have ever fully understoof the altter! $\endgroup$
    – Derek Holt
    Jan 14, 2014 at 8:58
3
$\begingroup$

This is a variant of Derek Holt's answer. Note that if $X$ is a finite group, and $S = \{x_{1},x_{2},\ldots,x_{r} \}$ is an irredundant generating set for $X$ ( that is, if any $x_{i}$ is omitted, the remaining elements do not generate $X$), then $\langle x_{1} \rangle < \langle x_{1},x_{2} \rangle < \ldots \langle x_{1},\ldots x_{r-1} \rangle < X$ is a strictly increasing sequence of subgroups of $X.$ It follows that $r \leq \log_{2}(|X|),$ in fact $r \leq \log_{p}(|X|),$ where $p$ is the smallest prime divisor of $|X|.$ Hence if $G$ is an infinite group, all of whose proper subgroups are finite, of order at most $n,$ then any irredundant generating set for $G$ has at most $1 + \log_{2}(n)$ elements.For if the chosen generating set has $s$ elements, then any $s-1$ elements irredundantly generate a proper subgroup, say $H$, of $G$, and $H$ has order at most $n.$ This bound can obviously be improved with finer knowledge of the possible prime divisors of orders of proper subgroups of $G.$

$\endgroup$

Your Answer

By clicking “Post Your Answer”, you agree to our terms of service and acknowledge you have read our privacy policy.

Not the answer you're looking for? Browse other questions tagged or ask your own question.